For random variables $(X_k)_{k=1}^infty$ in $mathbb{R}$, find $(c_k)_{k=1}^infty$ such that $P(lim...











up vote
0
down vote

favorite
1












I'm stuck with the following problem:



Let $(X_k)_{k=1}^infty$ be a sequence of real-valued random variables. Show that there is a real sequence $(c_k)_{k=1}^infty$ such that
$$Pleft(lim_{krightarrowinfty}frac{X_k}{c_k}=0right)=1.$$



With the typical approach for proving almost sure convergence through the Borel-Cantelli-lemma you arrive at
$$Pleft(lim_{krightarrowinfty}frac{X_k}{c_k}neq 0right)leq sum_{n=1}^infty Pleft(limsup_{krightarrowinfty}|X_k|>frac{c_k}{n}right)$$
and it would be sufficient to find $(c_k)_{k=1}^infty$ such that
$$sum_{k=1}^infty Pleft(|X_k|>frac{c_k}{n}right)<infty.$$
If the $X_k$ were identically distributed, I think a construction of the form $c_k=kcdot q_{1-1/(k^2)}$ could work, where $q_alpha$ is an $alpha$-quantile of $|X_1|$, but I can't do this in the general case.



How can I solve the general case (no further assumption about the $X_k$)? Is Borel-Cantelli even a good approach here?










share|cite|improve this question


























    up vote
    0
    down vote

    favorite
    1












    I'm stuck with the following problem:



    Let $(X_k)_{k=1}^infty$ be a sequence of real-valued random variables. Show that there is a real sequence $(c_k)_{k=1}^infty$ such that
    $$Pleft(lim_{krightarrowinfty}frac{X_k}{c_k}=0right)=1.$$



    With the typical approach for proving almost sure convergence through the Borel-Cantelli-lemma you arrive at
    $$Pleft(lim_{krightarrowinfty}frac{X_k}{c_k}neq 0right)leq sum_{n=1}^infty Pleft(limsup_{krightarrowinfty}|X_k|>frac{c_k}{n}right)$$
    and it would be sufficient to find $(c_k)_{k=1}^infty$ such that
    $$sum_{k=1}^infty Pleft(|X_k|>frac{c_k}{n}right)<infty.$$
    If the $X_k$ were identically distributed, I think a construction of the form $c_k=kcdot q_{1-1/(k^2)}$ could work, where $q_alpha$ is an $alpha$-quantile of $|X_1|$, but I can't do this in the general case.



    How can I solve the general case (no further assumption about the $X_k$)? Is Borel-Cantelli even a good approach here?










    share|cite|improve this question
























      up vote
      0
      down vote

      favorite
      1









      up vote
      0
      down vote

      favorite
      1






      1





      I'm stuck with the following problem:



      Let $(X_k)_{k=1}^infty$ be a sequence of real-valued random variables. Show that there is a real sequence $(c_k)_{k=1}^infty$ such that
      $$Pleft(lim_{krightarrowinfty}frac{X_k}{c_k}=0right)=1.$$



      With the typical approach for proving almost sure convergence through the Borel-Cantelli-lemma you arrive at
      $$Pleft(lim_{krightarrowinfty}frac{X_k}{c_k}neq 0right)leq sum_{n=1}^infty Pleft(limsup_{krightarrowinfty}|X_k|>frac{c_k}{n}right)$$
      and it would be sufficient to find $(c_k)_{k=1}^infty$ such that
      $$sum_{k=1}^infty Pleft(|X_k|>frac{c_k}{n}right)<infty.$$
      If the $X_k$ were identically distributed, I think a construction of the form $c_k=kcdot q_{1-1/(k^2)}$ could work, where $q_alpha$ is an $alpha$-quantile of $|X_1|$, but I can't do this in the general case.



      How can I solve the general case (no further assumption about the $X_k$)? Is Borel-Cantelli even a good approach here?










      share|cite|improve this question













      I'm stuck with the following problem:



      Let $(X_k)_{k=1}^infty$ be a sequence of real-valued random variables. Show that there is a real sequence $(c_k)_{k=1}^infty$ such that
      $$Pleft(lim_{krightarrowinfty}frac{X_k}{c_k}=0right)=1.$$



      With the typical approach for proving almost sure convergence through the Borel-Cantelli-lemma you arrive at
      $$Pleft(lim_{krightarrowinfty}frac{X_k}{c_k}neq 0right)leq sum_{n=1}^infty Pleft(limsup_{krightarrowinfty}|X_k|>frac{c_k}{n}right)$$
      and it would be sufficient to find $(c_k)_{k=1}^infty$ such that
      $$sum_{k=1}^infty Pleft(|X_k|>frac{c_k}{n}right)<infty.$$
      If the $X_k$ were identically distributed, I think a construction of the form $c_k=kcdot q_{1-1/(k^2)}$ could work, where $q_alpha$ is an $alpha$-quantile of $|X_1|$, but I can't do this in the general case.



      How can I solve the general case (no further assumption about the $X_k$)? Is Borel-Cantelli even a good approach here?







      probability convergence borel-cantelli-lemmas






      share|cite|improve this question













      share|cite|improve this question











      share|cite|improve this question




      share|cite|improve this question










      asked 13 hours ago









      Mau314

      36418




      36418






















          1 Answer
          1






          active

          oldest

          votes

















          up vote
          2
          down vote













          If $P{|X_k| >frac {c_k} {2^{k}}} <frac 1 {2^{k}}$ then $sum_k P{|X_k| >frac 1 {2^{k}}c_k} <infty$ and Borel Cantelli Lemma tells you that $frac {X_k} {c_k} to 0$ almost surely. You can always find $c_k$'s satisfying this condition, right? [All you need is the fact that $P{|X_k| >t} to 0$ as $t to infty$].






          share|cite|improve this answer























          • Yes, that's in spirit what I wanted to indicate with this quantile-construction (just that I compensate $n$ with $k$ rather than $2^k$) and now I don't know why I thought there's a problem with that. Thanks.
            – Mau314
            13 hours ago










          • I think you have a small typo: it should be $c_k/2^k$ in the first set
            – mathworker21
            11 hours ago










          • @mathworker21 Thanks. You are absolutely right.
            – Kavi Rama Murthy
            10 hours ago











          Your Answer





          StackExchange.ifUsing("editor", function () {
          return StackExchange.using("mathjaxEditing", function () {
          StackExchange.MarkdownEditor.creationCallbacks.add(function (editor, postfix) {
          StackExchange.mathjaxEditing.prepareWmdForMathJax(editor, postfix, [["$", "$"], ["\\(","\\)"]]);
          });
          });
          }, "mathjax-editing");

          StackExchange.ready(function() {
          var channelOptions = {
          tags: "".split(" "),
          id: "69"
          };
          initTagRenderer("".split(" "), "".split(" "), channelOptions);

          StackExchange.using("externalEditor", function() {
          // Have to fire editor after snippets, if snippets enabled
          if (StackExchange.settings.snippets.snippetsEnabled) {
          StackExchange.using("snippets", function() {
          createEditor();
          });
          }
          else {
          createEditor();
          }
          });

          function createEditor() {
          StackExchange.prepareEditor({
          heartbeatType: 'answer',
          convertImagesToLinks: true,
          noModals: true,
          showLowRepImageUploadWarning: true,
          reputationToPostImages: 10,
          bindNavPrevention: true,
          postfix: "",
          imageUploader: {
          brandingHtml: "Powered by u003ca class="icon-imgur-white" href="https://imgur.com/"u003eu003c/au003e",
          contentPolicyHtml: "User contributions licensed under u003ca href="https://creativecommons.org/licenses/by-sa/3.0/"u003ecc by-sa 3.0 with attribution requiredu003c/au003e u003ca href="https://stackoverflow.com/legal/content-policy"u003e(content policy)u003c/au003e",
          allowUrls: true
          },
          noCode: true, onDemand: true,
          discardSelector: ".discard-answer"
          ,immediatelyShowMarkdownHelp:true
          });


          }
          });














           

          draft saved


          draft discarded


















          StackExchange.ready(
          function () {
          StackExchange.openid.initPostLogin('.new-post-login', 'https%3a%2f%2fmath.stackexchange.com%2fquestions%2f2999437%2ffor-random-variables-x-k-k-1-infty-in-mathbbr-find-c-k-k-1-in%23new-answer', 'question_page');
          }
          );

          Post as a guest















          Required, but never shown

























          1 Answer
          1






          active

          oldest

          votes








          1 Answer
          1






          active

          oldest

          votes









          active

          oldest

          votes






          active

          oldest

          votes








          up vote
          2
          down vote













          If $P{|X_k| >frac {c_k} {2^{k}}} <frac 1 {2^{k}}$ then $sum_k P{|X_k| >frac 1 {2^{k}}c_k} <infty$ and Borel Cantelli Lemma tells you that $frac {X_k} {c_k} to 0$ almost surely. You can always find $c_k$'s satisfying this condition, right? [All you need is the fact that $P{|X_k| >t} to 0$ as $t to infty$].






          share|cite|improve this answer























          • Yes, that's in spirit what I wanted to indicate with this quantile-construction (just that I compensate $n$ with $k$ rather than $2^k$) and now I don't know why I thought there's a problem with that. Thanks.
            – Mau314
            13 hours ago










          • I think you have a small typo: it should be $c_k/2^k$ in the first set
            – mathworker21
            11 hours ago










          • @mathworker21 Thanks. You are absolutely right.
            – Kavi Rama Murthy
            10 hours ago















          up vote
          2
          down vote













          If $P{|X_k| >frac {c_k} {2^{k}}} <frac 1 {2^{k}}$ then $sum_k P{|X_k| >frac 1 {2^{k}}c_k} <infty$ and Borel Cantelli Lemma tells you that $frac {X_k} {c_k} to 0$ almost surely. You can always find $c_k$'s satisfying this condition, right? [All you need is the fact that $P{|X_k| >t} to 0$ as $t to infty$].






          share|cite|improve this answer























          • Yes, that's in spirit what I wanted to indicate with this quantile-construction (just that I compensate $n$ with $k$ rather than $2^k$) and now I don't know why I thought there's a problem with that. Thanks.
            – Mau314
            13 hours ago










          • I think you have a small typo: it should be $c_k/2^k$ in the first set
            – mathworker21
            11 hours ago










          • @mathworker21 Thanks. You are absolutely right.
            – Kavi Rama Murthy
            10 hours ago













          up vote
          2
          down vote










          up vote
          2
          down vote









          If $P{|X_k| >frac {c_k} {2^{k}}} <frac 1 {2^{k}}$ then $sum_k P{|X_k| >frac 1 {2^{k}}c_k} <infty$ and Borel Cantelli Lemma tells you that $frac {X_k} {c_k} to 0$ almost surely. You can always find $c_k$'s satisfying this condition, right? [All you need is the fact that $P{|X_k| >t} to 0$ as $t to infty$].






          share|cite|improve this answer














          If $P{|X_k| >frac {c_k} {2^{k}}} <frac 1 {2^{k}}$ then $sum_k P{|X_k| >frac 1 {2^{k}}c_k} <infty$ and Borel Cantelli Lemma tells you that $frac {X_k} {c_k} to 0$ almost surely. You can always find $c_k$'s satisfying this condition, right? [All you need is the fact that $P{|X_k| >t} to 0$ as $t to infty$].







          share|cite|improve this answer














          share|cite|improve this answer



          share|cite|improve this answer








          edited 10 hours ago

























          answered 13 hours ago









          Kavi Rama Murthy

          39k31748




          39k31748












          • Yes, that's in spirit what I wanted to indicate with this quantile-construction (just that I compensate $n$ with $k$ rather than $2^k$) and now I don't know why I thought there's a problem with that. Thanks.
            – Mau314
            13 hours ago










          • I think you have a small typo: it should be $c_k/2^k$ in the first set
            – mathworker21
            11 hours ago










          • @mathworker21 Thanks. You are absolutely right.
            – Kavi Rama Murthy
            10 hours ago


















          • Yes, that's in spirit what I wanted to indicate with this quantile-construction (just that I compensate $n$ with $k$ rather than $2^k$) and now I don't know why I thought there's a problem with that. Thanks.
            – Mau314
            13 hours ago










          • I think you have a small typo: it should be $c_k/2^k$ in the first set
            – mathworker21
            11 hours ago










          • @mathworker21 Thanks. You are absolutely right.
            – Kavi Rama Murthy
            10 hours ago
















          Yes, that's in spirit what I wanted to indicate with this quantile-construction (just that I compensate $n$ with $k$ rather than $2^k$) and now I don't know why I thought there's a problem with that. Thanks.
          – Mau314
          13 hours ago




          Yes, that's in spirit what I wanted to indicate with this quantile-construction (just that I compensate $n$ with $k$ rather than $2^k$) and now I don't know why I thought there's a problem with that. Thanks.
          – Mau314
          13 hours ago












          I think you have a small typo: it should be $c_k/2^k$ in the first set
          – mathworker21
          11 hours ago




          I think you have a small typo: it should be $c_k/2^k$ in the first set
          – mathworker21
          11 hours ago












          @mathworker21 Thanks. You are absolutely right.
          – Kavi Rama Murthy
          10 hours ago




          @mathworker21 Thanks. You are absolutely right.
          – Kavi Rama Murthy
          10 hours ago


















           

          draft saved


          draft discarded



















































           


          draft saved


          draft discarded














          StackExchange.ready(
          function () {
          StackExchange.openid.initPostLogin('.new-post-login', 'https%3a%2f%2fmath.stackexchange.com%2fquestions%2f2999437%2ffor-random-variables-x-k-k-1-infty-in-mathbbr-find-c-k-k-1-in%23new-answer', 'question_page');
          }
          );

          Post as a guest















          Required, but never shown





















































          Required, but never shown














          Required, but never shown












          Required, but never shown







          Required, but never shown

































          Required, but never shown














          Required, but never shown












          Required, but never shown







          Required, but never shown







          Popular posts from this blog

          Quarter-circle Tiles

          build a pushdown automaton that recognizes the reverse language of a given pushdown automaton?

          Mont Emei